sloan_galler
Thanks Received: 0
Forum Guests
 
Posts: 6
Joined: December 01st, 2010
 
 
 

Q6 - Recent investigations of earthquakes

by sloan_galler Mon Dec 06, 2010 7:01 pm

Can someone please explain why D is incorrect? I understand why B is correct, but I narrowed it down to B and D ...

My logic was is there is a range of displacement pulse intensity for earthquakes then is is very well possible that there was little significant damage ...

Thanks in advance!
 
giladedelman
Thanks Received: 833
LSAT Geek
 
Posts: 619
Joined: April 04th, 2010
 
This post thanked 2 times.
 
 

Re: Q6 - Recent investigations of earthquakes

by giladedelman Thu Dec 09, 2010 1:54 am

Thanks for your question!

We're trying to find an explanation for the fact that there was little damage to high-rise buildings by recent earthquakes, even though computer models suggest that high-rises are especially vulnerable to displacement pulses.

(B) gets the job done because if computer models often fail when tested in the field, this would help explain why the real-world results didn't jibe with what the computers predicted.

(D) is no good because we're told in the stimulus that the recent earthquakes were powerful. So we don't care much about low- and medium-intensity earthquakes.

(A) is incorrect because the distance the pulses travel has no bearing on why the high-rises weren't damaged.

(C) is incorrect because whether the pulses have accompanied every earthquake doesn't help us figure out the high-rise issue.

(E) actually makes the paradox even more vexing. If computer models are usually accurate, then, shoot, why didn't they work this time?

Does that answer your question?
 
jk8451
Thanks Received: 0
Forum Guests
 
Posts: 3
Joined: August 30th, 2011
 
 
 

Re: Q6 - Recent investigations of earthquakes

by jk8451 Sat Sep 03, 2011 5:37 pm

Hi there, I chose D) as well. I'm just wondering what is the difference between "Powerful" (as in the stimulus) and "The strongest" (as in D). It seems here they are equivalent to each other, but I don't think they really are in common sense... for example, if there are rank 1 to 10 for earth quake strengths, then 10 would be the "strongest", but it can be argued that all 7, 8, 9, 10 earth quakes are powerful.

As for B), I knew it does do the job, but somehow I got the weird feeling that the explanation i was too "irresponsible"... Does anyone else feel the same way?

I think I must have fallen into another LSAC trap...
 
giladedelman
Thanks Received: 833
LSAT Geek
 
Posts: 619
Joined: April 04th, 2010
 
This post thanked 1 time.
 
 

Re: Q6 - Recent investigations of earthquakes have

by giladedelman Wed Sep 07, 2011 5:42 pm

Right, "powerful" and "strongest" are not the same thing, but since the answer choice says that low- and medium-intensity earthquakes make pulses that are "much less powerful," it's safe to say those ones don't count as powerful. At any rate, this whole answer is out of scope because the stimulus doesn't say anything about earthquakes of different intensities.
 
aerialstrong
Thanks Received: 0
Forum Guests
 
Posts: 11
Joined: August 26th, 2012
 
 
 

Re: Q6 - Recent investigations of earthquakes

by aerialstrong Sun Aug 26, 2012 9:51 pm

It's not about "powerful'. i think this is out of scope coz there's no comparison in the argument nor premises. if the premises stated "one can only find damage to high-rises under strong earthquakes which carry strong displacement pulses", the D shall be right.
 
shirando21
Thanks Received: 16
Atticus Finch
Atticus Finch
 
Posts: 280
Joined: July 18th, 2012
 
 
 

Re: Q6 - Recent investigations of earthquakes

by shirando21 Fri Aug 31, 2012 8:16 pm

I picked D too, but in review of this question, I found if we follow the logic of D, in order to make it work, we are assuming that the recent earthquakes are low and median-intensity earthquakes, which we really shouldn't be assuming, it is like jumping ahead too much.
User avatar
 
LolaC289
Thanks Received: 21
Elle Woods
Elle Woods
 
Posts: 92
Joined: January 03rd, 2018
 
 
 

Re: Q6 - Recent investigations of earthquakes

by LolaC289 Tue Jul 24, 2018 3:02 am

I didn't expect the discussion on this question to be so short...I was most confused by this question in the whole section. Although I later found out I skipped over a very important phrase in the stimulus, I still think (B) is a terrible answer.

The thing that I skipped over is "Alarmingly, high-rise buildings are especially vulnerable......according to computer models." So when I was reviewing it, I kind of understand why (B) made some sense, because it points out that computer models may not be completely accurate in predicting things. Maybe as scientists use computer models to simulate the displacement pulse and high-rise buildings and investigate the effects the former have on the latter, some part of the models doesn't reflect the reality accurately. Maybe in real life, the buildings are much more solid than what appears in the models and thus the actual effect of the displacement pulse is exaggerated by the computer models.

However, I really don't like the way (B) delivered it. Isn't it exactly the reason why "scientific predictions based on computer models often fail when test in the field" that we are looking for? For me, (B) only points out the paradox without contributing anything to solve it. How can it be the correct answer? (Although, the rest choices are just as terrible as hell).

Please help with this one!
 
YurikaC738
Thanks Received: 0
Vinny Gambini
Vinny Gambini
 
Posts: 12
Joined: February 03rd, 2023
 
 
 

Re: Q6 - Recent investigations of earthquakes

by YurikaC738 Thu Apr 06, 2023 11:31 pm

LolaC289 Wrote:I didn't expect the discussion on this question to be so short...I was most confused by this question in the whole section. Although I later found out I skipped over a very important phrase in the stimulus, I still think (B) is a terrible answer.

The thing that I skipped over is "Alarmingly, high-rise buildings are especially vulnerable......according to computer models." So when I was reviewing it, I kind of understand why (B) made some sense, because it points out that computer models may not be completely accurate in predicting things. Maybe as scientists use computer models to simulate the displacement pulse and high-rise buildings and investigate the effects the former have on the latter, some part of the models doesn't reflect the reality accurately. Maybe in real life, the buildings are much more solid than what appears in the models and thus the actual effect of the displacement pulse is exaggerated by the computer models.

However, I really don't like the way (B) delivered it. Isn't it exactly the reason why "scientific predictions based on computer models often fail when test in the field" that we are looking for? For me, (B) only points out the paradox without contributing anything to solve it. How can it be the correct answer? (Although, the rest choices are just as terrible as hell).

Please help with this one!


For this particular question,
Expected: High-rising buildings within cities damaged by powerful earthquakes exposed to high damage to these structures (as predicted by computer models)
Unexpected: High-rises within cities indicates little damage
The correct answer would be able to explain why there's a difference between the expected and unexpected. B does the best job. Saying "scientific predictions based on computer models often fail when tested in the field", it means that the expected (in test) an go wrong in the field, and that is exactly where we at.

Honestly I am trapped by A. I thought traveling long distances by displacement pulses would make high-rises WITHIN CITIES not damaged (because it travels far more, probably to neighboring country but not somewhere closer lol). However, if so, it still does not explain why there's a discrepancy between the prediction and the reality (it would require us additionally assuming that the computer models too do not take that into account).